Inguinal Hernia Flashcards

1
Q

A 25/M undergoes emergency surgery for an incarcerated inguinal hernia. Following induction of anesthesia and just as soon as the inguinal incision is made, the incarcerated organ slips back into the abdominal cavity. In this patient:

A. The inguinal incision should be closed and a laparotomy performed.

B. The inguinal surgery should proceed and an attempt at retrieving the incarcerated organ made by cutting through the internal ring.

C. The hernia repair should proceed and when completed, a laparotomy performed.

D. If the fluid in the sac is clear and straw-colored, assume that the incarcerated organ is viable and only the groin surgery is necessary.

A

D.

If the fluid in the sac is clear and straw-colored, assume that the incarcerated organ is viable and only the groin surgery is necessary

How well did you know this?
1
Not at all
2
3
4
5
Perfectly
2
Q

A 15-year old female presents with a 2 year history of a non-reducible R inguinal mass. The mass is found to be firm, movable, non-reducible with no skin discoloration on physical examination. Abdominal findings were normal but the patient has fever and weight loss. What would be your most likely consideration?

A. Non-specific lymphadenitis

B. Incarcerated indirect inguinal hernia

C. Strangulated indirect inguinal hernia

D. Lymphoma

A

D. Lymphoma

How well did you know this?
1
Not at all
2
3
4
5
Perfectly
3
Q

A 30/M has a reducible inguinal mass. Three hours prior to consult, the mass became non-reducible with the patient complaining of severe pain. Abdominal findings were normal. The mass is located at the R inguinal region and is noted to be tender, fixed with no skin discoloration. What is the initial approach for this patient?

A. Perform emergency herniorrhaphy

B. Start IV resuscitation

C. Do a midline laparotomy incision

D. Do taxis

A

B. Start IV resuscitation

How well did you know this?
1
Not at all
2
3
4
5
Perfectly
4
Q

A patient previoiusly diagnosed to have a right indirect inguinal hernia noticed that the mass has become non-reducible for a day. On PE, he has a T of 38.9 C, the mass is noted to be tender and warm. He has direct and rebound tenderness at the right lower quadrant of the abdomen. What would be your surgical approach?

A. Laparotomy

B. Inguinal herniorrhaphy

C. Laparoscopic surgery

D. Mesh herniorrhaphy

A

A. Laparotomy

How well did you know this?
1
Not at all
2
3
4
5
Perfectly
5
Q

During a laparoscopic examination of the deep surface of the lower anterior abdominal wall, an incidental finding was a herniation of omentum below the inguinal ligament and medial to the external iliac vein. You now diagnose the patient as having

A. An indirect inguinal hernia

B. A direct inguinal hernia

C. A femoral hernia

D. An obturator hernia

A

C. A femoral hernia

How well did you know this?
1
Not at all
2
3
4
5
Perfectly
6
Q

Type of hernia with an indirect and direct hernia component?

A

Pantaloon hernia

How well did you know this?
1
Not at all
2
3
4
5
Perfectly
7
Q

Most common hernia:

A

Indirect inguinal hernia

How well did you know this?
1
Not at all
2
3
4
5
Perfectly
8
Q

What is your differential diagnosis for a groin mass?

A

Abscess

Epidermal inclusion cyst

Femoral artery aneurysm

Hernia

Hydrocele

Hematoma

Seroma

Lymphadenopathy

Sarcoma

Testicular torsion, and

Undescended testicle

How well did you know this?
1
Not at all
2
3
4
5
Perfectly
9
Q

On what side do inguinal hernias usually occur?

A

Right side

(delay in atrophy of processus vaginalis, after slower descent of right testis to scrotum during fetal development)

How well did you know this?
1
Not at all
2
3
4
5
Perfectly
10
Q

On what side do femoral hernias usually occur?

A

Right side

(possibly from tamponading effect of sigmoid colon on left femoral canal)

How well did you know this?
1
Not at all
2
3
4
5
Perfectly
11
Q

What type of hernia are most strangulated hernias?

A

Indirect inguinal hernia

How well did you know this?
1
Not at all
2
3
4
5
Perfectly
12
Q

What ligament is formed by the periosteum and fascia along the superior pubic ramus?

A

Cooper ligament

How well did you know this?
1
Not at all
2
3
4
5
Perfectly
13
Q

What attaches the testicle to the scrotum?

A

The gubernaculum

How well did you know this?
1
Not at all
2
3
4
5
Perfectly
14
Q

Name the contents of the spermatic cord:

A

Cremasteric muscle fibers

Testicular artery

Testicular pampiniform venous plexus

Genital branch of the genitofemoral nerve

Vas deferens, and

Processus vaginalis ± hernia sac

How well did you know this?
1
Not at all
2
3
4
5
Perfectly
15
Q

What are the boundaries of the inguinal canal?

A

Anterior: External oblique aponeurosis

Posterior: Transversalis fascia and the aponeurosis of transversus abdominis

Superior: Internal oblique and transversus abdominis musculoaponeurosis

Inferior: Inguinal ligament and lacunar ligament

How well did you know this?
1
Not at all
2
3
4
5
Perfectly
16
Q

Where do direct inguinal hernias occur with regard to Hesselbach’s triangle?

A

Direct hernias occur within Hesselbach’s triangle.

How well did you know this?
1
Not at all
2
3
4
5
Perfectly
17
Q

Where do indirect inguinal hernias occur with regard to Hesselbach’s triangle?

A

Indirect inguinal hernias occur lateral to Hesselbach’s triangle.

How well did you know this?
1
Not at all
2
3
4
5
Perfectly
18
Q

What nerve runs anterior to the spermatic cord in the inguinal canal and branches at the superficial inguinal ring?

A

llioinguinal nerve

How well did you know this?
1
Not at all
2
3
4
5
Perfectly
19
Q

What nerve innervates the skin on the lateral side of the scrotum and labia and the cremaster muscle?

A

The genital branch of the genitofemoral nerve

How well did you know this?
1
Not at all
2
3
4
5
Perfectly
20
Q

Which nerves provide sensation to the base of the penis, skin of the groin, and ipsilateral upper medial thigh?

A

The iliohypogastric and ilioinguinal nerves

How well did you know this?
1
Not at all
2
3
4
5
Perfectly
21
Q

What are the boundaries of the femoral canal?

A

Superior: Iliopubic tract

Inferior: Cooper ligament

Lateral: Femoral vein

Medial: Junction of iliopubic tract and Cooper ligament (lacunar ligament)

How well did you know this?
1
Not at all
2
3
4
5
Perfectly
22
Q

What kind of femoral hernias need to be repaired?

A

All femoral hernias need to be repaired (high incidence of strangulation).

How well did you know this?
1
Not at all
2
3
4
5
Perfectly
23
Q

What are the various methods through which a femoral hernia can be repaired?

A

Cooper ligament repair

(via a preperitoneal approach, or via a laparoscopic approach)

How well did you know this?
1
Not at all
2
3
4
5
Perfectly
24
Q

What are the essential steps of femoral hernia repair?

A

1) Dissection and reduction of the hernia sac.
2) Obliteration of the femoral canal defect in the femoral canal, by approximation of the iliopubic tract to the Cooper ligament versus placement of a prosthetic mesh.

How well did you know this?
1
Not at all
2
3
4
5
Perfectly
25
Q

What is the primary danger associated with a sliding hernia?

A

Failure to recognize the visceral component of the hernia sac before injury to bowel/ bladder.

How well did you know this?
1
Not at all
2
3
4
5
Perfectly
26
Q

Most common organ found in a sliding hernia in a female patient:

A

Ovaries/fallopian tubes

How well did you know this?
1
Not at all
2
3
4
5
Perfectly
27
Q

Most common organ found in a sliding hernia in a male patient:

A

Cecum/sigmoid colon

How well did you know this?
1
Not at all
2
3
4
5
Perfectly
28
Q

Next step if you cannot reduce an incarcerated femoral hernia:

A

Divide the inguinal or lacunar ligament

How well did you know this?
1
Not at all
2
3
4
5
Perfectly
29
Q

What should you do if during repair of an incarcerated inguinal hernia, with a concern for strangulation, you lose control of the hernia sac; and the contents of the hernia drop back into the abdomen?

A

Explore through preperitoneal incision by opening the peritoneum, versus laparoscopy.

How well did you know this?
1
Not at all
2
3
4
5
Perfectly
30
Q

What procedure do you perform to repair an inguinal hernia in infants and children?

A

High ligation of the hernia sac

How well did you know this?
1
Not at all
2
3
4
5
Perfectly
31
Q

What is the percentage risk for hernia incarceration with watchful waiting?

A

0.03%

How well did you know this?
1
Not at all
2
3
4
5
Perfectly
32
Q

What does a cord lipoma represent?

A

Retroperitoneal fat that has herniated through the deep inguinal ring

How well did you know this?
1
Not at all
2
3
4
5
Perfectly
33
Q

How many centimeters of mobilization can be obtained when component separation is applied to both sides of the abdominal wall?

A

Up to 20 cm of mobilization

How well did you know this?
1
Not at all
2
3
4
5
Perfectly
34
Q

What are the borders of the Triangle of Doom?

A

Vas deferens medially;

Spermatic vessels laterally;

External iliac vessels inferiorly

How well did you know this?
1
Not at all
2
3
4
5
Perfectly
35
Q

What structures are contained within the Triangle of Doom?

A

External iliac vessels

Deep circumflex iliac vein

Genital branch of genitofemoral nerve

Femoral nerve

How well did you know this?
1
Not at all
2
3
4
5
Perfectly
36
Q

What are the borders for the Triangle of Pain?

A

Spermatic vessels medially

Iliopubic tract superolaterally

How well did you know this?
1
Not at all
2
3
4
5
Perfectly
37
Q

What structures are contained within the Triangle of Pain?

A

Lateral femoral cutaneous nerve and anterior femoral cutaneous nerve of the thigh

How well did you know this?
1
Not at all
2
3
4
5
Perfectly
38
Q

Why is the Triangle of Pain significant?

A

Placing tacks in this area may lead to chronic pain from injury to the lateral femoral cutaneous nerve/anterior femoral cutaneous nerve of the thigh.

How well did you know this?
1
Not at all
2
3
4
5
Perfectly
39
Q

What is the Circle of Death in regard to hernia repair?

A

Also known as the corona mortis, a vascular ring formed by the joining of an aberrant artery with the normal obturator artery arising from a branch of the internal iliac artery; during a laparoscopic hernia repair, this vessel can be torn from both ends and bleed profusely.

How well did you know this?
1
Not at all
2
3
4
5
Perfectly
40
Q

What must you remember to do before leaving the operating room after repairing an inguinal hernia in a male patient?

A

Pull the testicle back down into the scrotum.

How well did you know this?
1
Not at all
2
3
4
5
Perfectly
41
Q

Most common early complication following hernia repair:

A

Urinary retention

How well did you know this?
1
Not at all
2
3
4
5
Perfectly
42
Q

Overall complication rate from hernia repair:

A

10%

How well did you know this?
1
Not at all
2
3
4
5
Perfectly
43
Q

Risk for surgical-site infection with open hernia repair:

A

Estimated to be 1% to 2%

How well did you know this?
1
Not at all
2
3
4
5
Perfectly
44
Q

Do you need to administer routine preoperative antibiotics to patients undergoing hernia repair?

A

No; considered a clean operation.

How well did you know this?
1
Not at all
2
3
4
5
Perfectly
45
Q

Which patients undergoing hernia repair might you consider giving preoperative antibiotics to?

A

Patients with significant underlying disease (ASA score >III)

How well did you know this?
1
Not at all
2
3
4
5
Perfectly
46
Q

If you have to give preoperative antibiotics before a hernia repair, what antibiotics do you give? How about a penicillin-allergic patient?

A

Cefazolin, 1 to 2g IV 30 to 60 minutes before incision;

If penicillin allergic, give Clindamycin 600mg IV or Vancomycin 1g IV.

How well did you know this?
1
Not at all
2
3
4
5
Perfectly
47
Q

What nerves are most commonly affected during open hernia repair?

A

Ilioinguinal nerve

Iliohypogastric nerve

Genital branch of the genitofemoral nerve

How well did you know this?
1
Not at all
2
3
4
5
Perfectly
48
Q

What nerves are most commonly affected during laparoscopic hernia repair?

A

Lateral femoral cutaneous and genitofemoral nerves

How well did you know this?
1
Not at all
2
3
4
5
Perfectly
49
Q

What is the usual best treatment for patients who present with a nerve entrapment syndrome after open inguinal hernia repair?

A

Initial treatment should include a long trial of observation, NSAIDS, and possibly nerve blocks (exhaust every possible solution before taking the patient back, unless the patient is experiencing nerve entrapment immediately postoperatively in the PACU; in that case, take them back).

Repeat exploration with neurectomy and mesh removal as needed.

How well did you know this?
1
Not at all
2
3
4
5
Perfectly
50
Q

What is the pathogenesis of ischemic orchitis after inguinal hernia repair?

A

Thrombosis of the small veins of the pampiniform plexus within the spermatic cord resulting in venous congestion of the testis with resultant progressive pain and swelling usually resulting in testicular atrophy (the most common cause is extensive dissection of a scrotal hernia sac).

How well did you know this?
1
Not at all
2
3
4
5
Perfectly
51
Q

Where does recurrence of an inguinal hernia usually occur?

A

The floor of the inguinal canal near the pubic tubercle where the tension on the suture line is the greatest.

How well did you know this?
1
Not at all
2
3
4
5
Perfectly
52
Q

What is the rate of hernia recurrence?

A

1% to 3%

How well did you know this?
1
Not at all
2
3
4
5
Perfectly
53
Q

What is the time period that hernias usually recur?

A

Usually within the first 2 years after repair

How well did you know this?
1
Not at all
2
3
4
5
Perfectly
54
Q

How would you repair an incisional hernia with a small defect (<2 cm in diameter) with viable surrounding tissue?

A

Primary repair

How well did you know this?
1
Not at all
2
3
4
5
Perfectly
55
Q

When would you use a prosthetic mesh to repair an incisional hernia?

A

Incisional hernia with a large defect (>2-3 cm diameter)

How well did you know this?
1
Not at all
2
3
4
5
Perfectly
56
Q

What is the minimal desired overlap of mesh around the defect when performing a laparoscopic ventral hernia repair?

A

4cm

How well did you know this?
1
Not at all
2
3
4
5
Perfectly
57
Q

What kind of hernia occurs at sites where vessels and nerves perforate the linea alba?

A

Epigastric hernia

How well did you know this?
1
Not at all
2
3
4
5
Perfectly
58
Q

What kind of hernia occurs between the rectus muscle medially and the semilunar line laterally?

A

Spigelian hernia

How well did you know this?
1
Not at all
2
3
4
5
Perfectly
59
Q

Why is it recommended that spigelian hernias be repaired?

A

The risk for incarceration associated with its relatively narrow neck.

How well did you know this?
1
Not at all
2
3
4
5
Perfectly
60
Q

What is the Howship-Romberg sign?

A

Obturator hernia causing pain along the medial aspect of the proximal thigh from nerve compression.

How well did you know this?
1
Not at all
2
3
4
5
Perfectly
61
Q

What are the borders of the superior lumbar (Grynfeltt) triangle?

A

Medial: Quadratus lumborum

Lateral: Internal abdominal oblique

Superior: 12th rib

Floor: Transversalis fascia

Roof: External abdominal oblique

How well did you know this?
1
Not at all
2
3
4
5
Perfectly
62
Q

What are the borders of the inferior lumbar (Petit) triangle?

A

Medial: Latissimus dorsi

Lateral: External abdominal oblique

Inferior: Iliac crest

(Floor: Internal abdominal oblique)

How well did you know this?
1
Not at all
2
3
4
5
Perfectly
63
Q

Which lumbar hernia is more common: Grynfeltt hernia or Petit hernia?

A

Grynfeltt hernia

How well did you know this?
1
Not at all
2
3
4
5
Perfectly
64
Q

The most common symptom of a sciatic hernia:

A

Presence of an uncomfortable or slowly enlarging mass in the gluteal or intragluteal area

How well did you know this?
1
Not at all
2
3
4
5
Perfectly
65
Q

What options do you have after an inadvertent enterotomy during an elective hernia repair (after repair of the enterotomy)?

A

Abort the hernia repair;

Perform a primary tissue or biologic tissue repair;

Perform a delayed repair using prosthetic mesh in 3 to 4 days.

How well did you know this?
1
Not at all
2
3
4
5
Perfectly
66
Q

What is an Amyand hernia?

A

Hernia sac containing a ruptured appendix.

How well did you know this?
1
Not at all
2
3
4
5
Perfectly
67
Q

What is a Bochdalek hernia?

A

A posterior diaphragmatic hernia usually occurring on the left (Bochdalek = back and to the left)

How well did you know this?
1
Not at all
2
3
4
5
Perfectly
68
Q

What is a Morgagni hernia?

A

An anterior parasternal diaphragmatic hernia

69
Q

What is a Cooper hernia?

A

A femoral hernia with 2 sacs that tracks into the scrotum or labia majorum through the femoral canal

70
Q

What is a Littre hernia?

A

Hernia containing a Meckel diverticulum

71
Q

What is a Spigelian hernia?

A

Hernia through the linea semilunaris

72
Q

What is a Hesselbach hernia?

A

A hernia that is lateral to the femoral vessels and under the inguinal ligament.

73
Q

What is a pantaloon hernia?

A

A hernia that straddles the inferior epigastric vessels representing both a direct hernia through the floor of the canal and an indirect hernia protruding through the internal ring

74
Q

What is a Richter hernia?

A

Incarcerated/strangulated hernia involving only 1 sidewall of the bowel

75
Q

What is an intraparietal hernia?

A

Hernia containing abdominal contents occurring between the layers of the abdominal wall

76
Q

What is a sports hernia?

A

Characterized by a dilated superficial ring of the inguinal canal and chronic groin pain in athletes (not a true hernia)

77
Q

Name the repair:

Approximation of the transversus abdominis aponeurotic arch to the iliopubic tract, with the use of interrupted sutures; beginning at the pubic tubercle and extending laterally past the internal inguinal ring with or without the use of a relaxing incision.

A

The iliopubic tract

78
Q

Name the repair:

Suturing the transversus abdominis and internal oblique musculoaponeurotic arches or conjoined tendon to the inguinal ligament

A

The Bassini repair

79
Q

Name the repair:

A multilayer imbricated repair of the posterior wall of the inguinal canal with a continuous running suture technique by superimposing running suture lines progressing from deep to more superficial layers (initial suture line-transversus abdominis aponeurotic arch to iliopubic tract, next suture line-internal oblique and transversus abdominis muscles and aponeuroses to the inguinal ligament)

A

The Shouldice repair

80
Q

Name the repair:

The edge of the transversus abdominis aponeurosis is approximated to Cooper ligament with interrupted, nonabsorbable sutures; a transition suture is then placed to incorporate the Cooper ligament and the iliopubic tract when the medial aspect of the femoral canal is reached; the transversus abdominis aponeurosis is then secured to the iliopubic tract lateral to the transition stitch; a relaxing incision is then made throughout the extent of the anterior sheath to near its lateral border

A

McVay (Cooper ligament) repair

81
Q

Name the tension-free inguinal hernia repair:

Prosthetic nonabsorbable mesh is sutured to the aponeurotic tissue overlying the pubic bone medially continuing along the transversus abdominis or conjoined tendon superiorly and the iliopubic tract or the shelving edge of the inguinal (Poupart) ligament inferolaterally using nonabsorbable monofilament suture in a continuous fashion

A

Lichtenstein repair

82
Q

Name the tension-free inguinal hernia repair:

A cone-shaped plug of polypropylene is inserted into the internal inguinal ring and sewn to the surrounding tissues, occluding the hernia, and held in place by an overlying mesh patch (with or without sutures)

A

Plug and patch repair

83
Q

Name the tension-free inguinal hernia repair:

A repair that uses a bilayered device with 3 polypropylene components (underlay circular patch, connector, and onlay patch) covers the posterior inguinal floor

A

Prolene hernia system (PHS) repair

84
Q

Name the hernia repair:

A hernia repair that utilizes an infraumbilical incision and blunt dissection to create a space beneath the rectus with placement of a dissecting balloon deep to the posterior rectus sheath and advanced to the pubic symphysis and inflated under direct laparoscopic vision

A

Totally extraperitoneal (TEP) repair

85
Q

Name the hernia repair:

A hernia repair that utilizes an infraumbilical incision to gain access to the peritoneal cavity directly; placement of two 5-mm ports lateral to the inferior epigastric vessels at the level of the umbilicus; creation of a peritoneal flap

A

Transabdominal preperitoneal (TAPP) repair

86
Q

Name the hernia repair:

A large piece of mesh is placed with an extensive fasdal underlay in the retromuscular space on top of either the posterior rectus sheath or peritoneum

A

Stoppa repair

87
Q

Name the hernia repair:

Large subcutaneous flaps above the external oblique fascia are created; a relaxing incision is performed on the lateral external oblique aponeurosis from the costal margin to the pubis; the external oblique is then bluntly dissected from the internal oblique with the option of performing further relaxing incisions (aponeurotic layers of the internal oblique, transversus abdominis, or posterior rectus sheath); primary fascial closure at midline. These techniques, when applied to both sides of the abdominal wall, can yield up to 20cm of mobilization.

A

Component separation technique

88
Q

Which of the following is the most common location for recurrence at reoperation following laparoscopic repair of an inguinal hernia?

A. Anterior

B. Posterior

C. Medial

D. Lateral

A

C

The most common location for recurrence following laparoscopic hernia repair is the medial aspect of the mesh.

This relates to failure to secure the mesh medially and using a piece of mesh that is too small.

With a large direct defect, it is important to have adequate coverage anteriorly or the mesh can prolapse into the hernia defect.

89
Q

A 55-year-old female presents with a tender mass below the inguinal ligament. During repair of the femoral hernia through an inguinal approach, you are unable to reduce the bowel. Which of the following is the next best step?

A. Push up on the bowel inferiorly while pulling up on the bowel superiorly

B. Perform a small segmental bowel resection

C. Divide the inguinal ligament

D. Make a small contralateral inguinal incision to place a laparoscope

A

C

If you are unable to reduce the bowel during a femoral hernia repair through an inguinal approach, the next best step is to incise the inguinal ligament.

Make sure to control the neck of the hernia so that you can ensure viability of the bowel and then repair the hernia and inguinal ligament.

90
Q

Which of the following is the most common presentation of an obturator hernia?

A. Bowel obstruction

B. Dysuria

C. Testicular torsion

D. Paresthesias of the lateral thigh

A

A

Obturator hernias are very rare but occur more frequently in multiparous elderly women.

They can be associated with the Howship-Romberg sign, which is pain on internal rotation of the thigh.

An obturator hernia occurs through a weakened obturator membrane. The defect is usually narrow in diameter, which predisposes to small-bowel incarceration and strangulation.

91
Q

In which of the following patients is laparoscopic hernia repair indicated?

A. 65-year-old male with bilateral inguinal hernias with a prior radical retropubic prostatectomy

B. 37-year-old pregnant female in the second trimester with an incarcerated left inguinal hernia with a previous caesarean section

C. 45-year-old female with bilateral recurrent inguinal hernias after laparoscopic repair who recently underwent pelvic radiation

D. 27-year-old male with a history oflaparoscopic cholecystectomy with bilateral inguinal hernias

A

D

Laparoscopic inguinal hernia repair has advantages in the setting of bilateral and/or recurrent hernias.

Contraindications include patients who have had prior laparoscopic hernia repair, prior preperitoneal pelvic surgery or previous lower midline incision, prior pelvic radiation, or cannot tolerate general anesthesia.

92
Q

Which of the following is the most likely cause of ischemic orchitis following inguinal hernia repair?

A. Ligation of the testicular artery

B. Complete excision of a large scrotal hernia sac

C. Inadvertent torsion of the testicular cord during the repair

D. Reconstruction of an internal ring that causes compression of the testicular cord

A

B

Extensive dissection of the spermatic cord is the most common risk factor in the development of ischemic orchitis following inguinal hernia repair.

Ischemic orchitis likely results from testicular venous congestion from thrombosis of the pampiniform venous plexus.

To help prevent this, the spermatic cord should not be dissected past the pubic tubercle, and the hernia sac can be ligated and divided, leaving the distal sac in place.

Ligation of the testicular artery is usually well tolerated, as evidenced by the 2-stage Fowler-Stevens orchiopexy for high undescended testis, because the testicle has a rich collateral blood supply from the cremasteric artery (branch of inferior epigastric artery) and the artery to the ductus deferens (branch of inferior vesical artery).

93
Q

A 52-year-old chronic alcoholic male who has never been to a hospital presents with slow oozing of clear fluid through thin skin at the apex of his large umbilical hernia. He has a shifting dullness on physical examination. Which of the following is the next best step in management?

A. Immediate umbilical hernia repair with mesh

B. Perform paracentesis, primarily repair the skin defect, and place an abdominal binder

C. Treat with salt and fluid restriction, oral antibiotics, and place a negative-pressure wound dressing over the skin defect

D. Aggressive diuresis, IV antibiotics, and bed rest followed by hernia repair during the same hospital admission

A

D

Umbilical hernia repair in cirrhotic patients with uncontrolled ascites has a high morbidity and mortality rate.

Optimal therapy includes aggressive diuresis and sodium/fluid restriction to control ascites, IV antibiotics to combat possible bacterial peritonitis, and bed rest to reduce stress on the hernia and leaking site, followed by repair of the hernia after the ascites is controlled.

94
Q

Which of the following nerves is not at risk during a laparoscopic repair of an inguinal hernia?

A. Pudendal nerve

B. Femoral branch of the genitofemoral nerve

C. Lateral cutaneous nerve of the thigh

D. Intermediate cutaneous branch of the anterior branch of the femoral nerve

A

A

The pudendal nerve is not at risk of injury during laparoscopic hernia repair. To help reduce the incidence of nerve injury during laparoscopic hernia repair, placing tacks lateral to the epigastric vessels is avoided.

95
Q

Which of the following statements regarding umbilical hernia is true?

A. Early repair of umbilical hernias in infants should be performed because of the risk of incarceration

B. Umbilical hernias are more common in white infants than in African American infants

C. Repair of umbilical hernias in adults is usually indicated

D. Umbilical hernias in infants usually require surgery

A

C

Umbilical hernias in adults should usually be performed promptly because of the risk of incarceration.

Umbilical hernias are rarely incarcerate, and most surgeons will defer surgery until about 4 to 5 years of age because the majority of umbilical hernias in infants will close spontaneously by 2 years of age.

Umbilical hernias are about 8 times more common in African American children than in white children.

96
Q

Which of the following is the best choice for repairing a unilateral indirect inguinal hernia in a 5-month-old-male infant?

A. Open repair with PHS

B. High ligation of the hernia sac

C. Laparoscopic TEP hernia repair

D. Cooper ligament (McVay) repair

A

B

Inguinal hernia in infants results from a patent processus vaginalis and not from a weakness in the floor of the inguinal canal.

High ligation of the hernia sac is usually all that is required except in the instance of a large defect, which may require tightening of the internal ring or reconstruction of the inguinal floor.

97
Q

Which of the following statements is true regarding hernias?

A. Femoral hernias are more common in males than in females

B. Inguinal hernias occur more frequently in females than in males

C. Indirect inguinal hernias account for two-thirds of all inguinal hernias

D. Indirect hernias are a rare occurrence in women

A

C

Indirect inguinal hernias account for two-thirds of inguinal hernias.

Femoral hernias account for approximately 5% of abdominal wall hernias and occur more frequently in women.

Inguinal hernias are approximately 25 times more common in men than in women.

The most common hernia in both men and women is an indirect inguinal hernia.

98
Q

Part of the circumference of bowel is incarcerated or even strangulated within the fascial defect in this type of hernia:

A. Spigelian hernia

B. Amyand hernia

C. Richter hernia

D. Cooper hernia

A

C

Richter hernia occurs in the setting of other hernias and is not an anatomically distinct hernia type like the other listed hernias.

Part of the circumference of bowel is incarcerated or even strangulated within the fascial defect of any hernia in the viscera area.

99
Q

What is the best definition of the myopectineal orifice of Fruchaud?

A. A weakness of the transversalis fascia

B. A defect in the rectus abdominis

C. A primary defect of the conjoined tendon

D. Weakness at the semilunar line

A

A

All groin hernias protrude through the myopectineal orifice of Fruchaud (MPO), which is a primary weakness or defect in the transversalis fascia of the aponeurosis immediately outside the peritoneum.

100
Q

A 39-year-old male has a CT scan of the abdomen which shows a 8-cm anterior abdominal wall mass with a nodular, nonhomogeneous pattern of enhancement. What is the best first step for this patient?

A. Further radiographic characterization

B. Incisional biopsy to establish the diagnosis

C. Definitive surgical excision

D. Chemotherapy

A

B

The findings are suspicious for a sarcoma.

The first best step is incisional biopsy, especially in light of the size of the mass.

101
Q

This hernia repair is effective for femoral hernia but always requires a relaxing incision to relieve tension along the repair:

A. Bassini

B. Halsted

C. Mcvay

D. Shouldice

A

C

The McVay repair is effective for femoral hernia but always requires a relaxing incision to relieve tension.

Since it’s a nonmesh repair, the recurrence rate can be as high as 30%.

102
Q

Which is false about the boundaries of the space of Retzius (retropubic)?

A. Symphysis pubis anteriorly

B. Pubic rami laterally

C. Pubic bone makes up the sidewall

D. In front of the transversalis fascia

A

D

The space of Retzius (retropubic) lies behind the transversalis fascia and also the symphysis pubis lies anteriorly and pubic rami laterally, with the pubic bone making up the sidewall.

103
Q

Match the hernia with the description.

A. Richter hernia

B. Littre’s hernia

C. Lumbar hernia

D. Spigelian hernia

E. Amyand’s hernia

I: Hernia containing the appendix

II: Hernia containing the anti mesenteric wall of the intestine

III. Hernia in between the semilunar line and the rectus muscle

IV. Hernia containing a Meckel’s diverticulum

V. Hernia through the superior or inferior lumbar triangle

A

ANSWER: A-II; B-IV; C-V; D-III; E-I COMMENTS: A Richter hernia is an abdominal wall hernia that only contains the antimesenteric border of the small intestine. This type of hernia may be dangerous, because the incarcerated portion can strangulate without showing symptoms. A Littre’s hernia is an abdominal wall hernia containing a Meckel’s diverticulum. A lumbar hernia is a rare back hernia that can form through the superior or inferior lumbar triangles. The superior triangle has boundaries of the 12th rib, internal oblique muscle, and thoracic paraspinal muscle. The inferior triangle has boundaries of the iliac crest, external oblique muscle, and latissimus dorsi muscle. A spigelian hernia is a lateral abdominal wall hernia that forms between the semilunar line and the rectus muscle. These often form no noticeable bulge because the external oblique aponeurosis is intact. An Amyand’s hernia is an abdominal wall hernia that contains the appendix.

104
Q

Chronic groin pain following an inguinal hernia repair may be the result of:

A. Division of the nerves during the surgical procedure

B. Postoperative scar tissue

C. Use of a mesh

D. Injury from the use of tacks or staples

E. All of the above

A

ANSWER: E COMMENTS: Groin pain following an inguinal hernia repair is much more common than recurrence and has occurred at incidence as high as 29%–76% in several series. Transient pain with mild numbness inferior to the incision is common. Persistent, intense pain and loss of sensation suggest nerve injury or entrapment. In open repairs, the most common nerves involved are the ilioingui- nal, iliohypogastric, or genital branch of the genitofemoral nerve. Injury to the lateral femoral cutaneous and genitofemoral nerves in laparoscopic repairs may occur from tack placement. Mesh inguino- dynia, or post herniorraphy pain syndrome, has been reported to result from an inflammatory response to mesh or resultant scar tissue, or both. Although nerve blocks may be diagnostic or thera- peutic, exploration may be necessary along with neurectomy, surgi- cal removal of all or part of a nerve. To prevent the problem from occurring, the best intervention is to be meticulous with identification and avoid entrapment of the aforementioned nerves in open repairs. In addition, tack placement should be avoided, particularly in the triangle of doom in laparo- scopic procedures.

105
Q

Which of the following statements is true regarding surgical technique?

A. A Bassini repair can be used for femoral hernias.

B. A Shouldice repair approximates the transversus abdominis aponeurosis to Cooper’s ligament medially and the iliopubic tract laterally. It requires a relaxing incision.

C. A total extraperitoneal repair (TEP) is a laparoscopic approach that stays in the preperitoneal space by using a balloon dissector.

D. A Bassini repair uses a piece of mesh to reinforce the floor of the inguinal canal and recreate the internal ring in a tension-free manner.

E. A McVay repair may never be used for femoral hernias.

A

C

A McVay repair is described in answer E and can be used for femoral hernias. A transition stitch and a relaxing incision are needed for a McVay repair—medial to the edge of the femoral canal the transversus abdominis aponeurosis is stitched to Cooper’s ligament, and lateral to the femoral canal the transversus abdominis is attached to the iliopubic tract.

A Bassini repair is a tissue repair, and answer D describes a Lichtenstein repair. Since the Bassini repair is completely superior to the inguinal ligament, it cannot be used to repair a femoral hernia.

106
Q

Which of the following statements are true regarding laparoscopic inguinal hernia anatomy?

A. The triangle of doom is bordered by the vas deferens, iliopubic tract, and gonadal vessels and contains the ilioinguinal and iliohypogastric nerves that must be avoided to prevent pain.

B. The lateral-most border of the dissection is the anterior superior iliac spine.

C. The femoral branch of the genitofemoral nerve and the lateral femoral cutaneous nerve are located medial and superior to the iliopubic tract.

D. The femoral canal cannot be accessed through a preperitoneal approach; therefore femoral hernias cannot be repaired laparoscopically.

E. A transabdominal preperitoneal repair (TAPP) approach uses a dissecting balloon to stay in the preperitoneal space throughout the procedure, and a TEP approach initially accesses the peritoneum before creating a peritoneal flap.

A

B

Laparoscopic hernia repair is appropriate for femoral hernias and indicated for bilateral or recurrent inguinal hernias. There are two techniques to gain exposure to the inguinal region.

A TEP uses an infraumbilical port and a balloon dissector in the preperitoneal space to expose the inguinal canal and hernia.

A TAPP uses an infraumbilical port to gain access to the peritoneum. A peritoneal flap is created to expose the hernia.

The lateral edge of the dissection is the anterior superior iliac spine in both approaches.

The hernia is reduced, and a large mesh is placed over the defect.

Important landmarks are the inferior epigastric vessels running along the edge of the rectus muscle dividing indirect and direct hernias.

Tacks are placed medial to Cooper’s ligament to secure the mesh in place. No tacks should be placed inferior to the iliopubic tract because the femoral branch of the genitofemoral nerve and the lateral femoral cutaneous nerve are located lateral and inferior.

The triangle of pain contains the genitofemoral nerve and lateral cutaneous femoral nerve and is bound by the iliopubic tract superiorly and genital vessels inferiorly.

The triangle of doom contains the external iliac vessels, and the femoral nerve is bound by the iliopubic tract superiorly, vas deferens medially, and spermatic vessels laterally.

107
Q

Which of the following statements is false regarding the iliopubic tract?

A. It extends from the anterior superior iliac spine to the pubis.

B. It is a condensation of the transversalis fascia.

C. It is of anatomic interest but has little clinical significance.

D. It runs underneath the shelving portion of the Poupart’s ligament.

E. Many branches of the lumbar plexus run inferior to the iliopubic tract.

A

C

The transversalis fascia is the portion of the endo-abdominal fascia that underlies the transversus abdominis muscle. It has several thickenings, the most important of which is the iliopubic tract, which arises from the iliopectineal arch, inserts on the anterior superior iliac spine, and extends over the femoral vessels to the pubis.

Proper utilization of the transversalis fascia during the repair of an inguinal hernia is important to the success of operations not using the prosthetic material.

The iliopubic tract has particular significance because of its importance as a landmark to laparoscopic surgeons. Many of the branches of the lumbar plexus run inferior to the tract, and damage to these nerves may be the result of aggressive dissection or the placement of tacks or staples to affix a mesh below this structure.

108
Q

Which of the following statements is false regarding the incidence of abdominal wall hernias?

A. Two-thirds of all inguinal hernias are classified as indirect.

B. Femoral hernias are more common in females than in males.

C. Indirect hernias are common in females.

D. Hernias generally occur with equal frequency in males and females.

E. Premature babies have a 10% incidence of inguinal hernia.

A

D

Approximately three-fourths of all abdominal wall hernias occur in the inguinal region, and roughly two-thirds of them are indirect inguinal hernias.

Groin hernias are considered to be at least 25 times more common in males than in females.

The incidence of inguinal hernias is increased by prematurity.

The most common hernia in each gender is an indirect inguinal hernia.

Femoral hernias are rare in men. On the other hand, direct hernias are uncommon in women.

It has been estimated that inguinal hernias develop in 25% of males and 2% of females during their lifetime. Therefore hernias constitute a significant economic problem in terms of loss of time from work.

109
Q

Which of the following statements is true regarding direct inguinal hernias?

A. Direct hernias are commonly congenital and found in younger patients.

B. The risk of incarceration of a direct hernia is high.

C. A direct hernia is solely a weakening of the inguinal floor and does not pass through the inguinal rings.

D. An indirect hernia may be present as well.

E. An indirect hernia will never be present with it.

A

ANSWER: D COMMENTS: Through physical stress, the connective tissue breaks down. As a result, the strength of the transverse aponeurosis and fascia decreases from intraabdominal pressure, smoking, aging, connective tissue disease, and systemic illnesses. Therefore direct inguinal hernias are acquired from the “wear and tear” of daily life, including straining to urinate or defecate, chronic coughing, and heavy lifting. A decrease in the content of hydroxyproline in the aponeurosis of patients with hernias has been demonstrated, along with alterations in the ultrastructure of collagen. Large indirect hernias may weaken the floor of the Hesselbach’s triangle and result in a functional direct component. Because the area of the Hesselbach’s triangle is weaker without a narrow-necked sac, the risk for incarceration is low. Rarely, incarceration results when a direct hernia passes through the external ring posterior to the cord structures. The spermatic cord should be explored to rule out the presence of an indirect sac. A pantaloon hernia is composed of both direct and indirect inguinal hernias.

110
Q

Which of the following statements about the management of inguinal hernias in infants and children is true?

A. Repair should be delayed until a child reaches school age since most inguinal hernia defects close spontaneously.

B. Repair usually requires a Bassini procedure.

C. The distal sac should be removed to prevent the formation of a secondary hydrocele.

D. Contralateral inguinal exploration is indicated routinely because of the high risk for bilaterality.

E. Intubation of the clinically apparent hernia sac with a laparoscope is one method of examining the contralateral side.

A

E

Inguinal hernias in infants and children are nearly always indirect and result from the failure of obliteration of the processus vaginalis.

Effective treatment requires only high ligation and transection of the sac with or without excision of the distal component.

Repair need not be delayed unless the infant has associated medical problems. In fact, bowel obstruction and gonadal or intestinal infarction as a result of strangulation are most likely to occur during the first 6 months of life. Therefore the repair should be performed soon after the diagnosis is made.

Exploration of the opposite side in children with a unilateral inguinal hernia is controversial. The incidence of a contralateral hernia following unilateral inguinal herniorrhaphy in children has been reported to be 10%–30%.

Contralateral exploration should be performed routinely in the subset of patients most likely to have a clinically occult hernia: children younger than 2 years, girls younger than 3 years (higher bilateral rate), patients with ventriculoperitoneal shunts, and children younger than 2 years with a left-sided hernia.

This last recommendation is based on the fact that most (60%) pediatric hernias are right sided.

Intubation of the clinically apparent hernia sac with a laparoscope is one method of examining the contralateral side.

111
Q

Which of the following hernias is most likely to recur after a primary repair?

A. Epigastric hernia

B. Spigelian hernia

C. Indirect hernia

D. Femoral hernia

E. Incisional hernia

A

E

The primary repair of incisional hernias can be associated with a 30%–50% or higher recurrence rate, depending on the size of the hernia. Except for small incisional hernias, a prosthetic mesh is necessary to reduce recurrence rates to 10% or possibly less.

Patients with incisional hernias usually have predisposing factors, such as obesity, chronic debilitating illness, diabetes, advanced age, and smoking. The predisposing factors also play a role in the failure of primary repair.

Recurrence rates after the other listed hernia repairs should all be 5% or less.

112
Q

Which of the following is a true statement regarding umbilical hernias?

A. They are the embryonic equivalent of a small omphalocele.

B. Repair in infants is usually deferred until approximately 1 year of age.

C. Repair in adults is generally indicated.

D. The “vest-over-pants” type of repair is stronger than simple approximation of fascial margins.

E. They are most common in white infants.

A

C

Umbilical hernias are the result of a patent umbilical ring, whereas an omphalocele is the result of the failure of abdominal wall closure in the midline during early intrauterine life.

Umbilical hernias are said to be present in 40%–90% of African- American infants.

Incarceration is rare in infants. Unless the defect is large, most surgeons defer repair until the child is approximately 4 years of age because spontaneous closure does occur. In adults, however, repair should be carried out promptly because of the risk for incarceration.

There is no convincing evidence that a “vest- over-pants” type of repair is structurally superior to a simple approximation of the fascial margins.

Repair in adults may benefit from the use of prosthetic material, such as polypropylene, if the fascial defect is large or tension is present. There are several variations of mesh repairs with no evidence-based consensus of choice at this time.

113
Q

A 75-year-old man is seen in the emergency department with a 2-h history of an incarcerated femoral hernia. He takes warfarin for a past history of atrial fibrillation and has an international normalized ratio (INR) of 3.1. Which of the following is the correct treatment?

A. Admit the patient for correction of the INR and repair the hernia in the morning.

B. Perform an emergency laparoscopic repair of the hernia.

C. Perform an emergency open repair of the hernia.

D. Attempt a reduction of the hernia in the emergency department after sedation.

E. Use a mesh in the repair of the hernia.

A

ANSWER: C COMMENTS: The risk for strangulation in incarcerated femoral hernias is reported to be as high as 20%–40%. It is believed that the window for successful treatment to avoid bowel resection is 4 to 6 h. This clinical condition is considered a surgical emergency, and fresh-frozen plasma can be administered just before and during surgery. A laparoscopic repair can be challenging with an incarcer- ated hernia, and in addition there is a concern for increased bleeding in the patient. Laparoscopic repair also requires the use of a mesh, which may become infected. Although mesh decreases recurrence in femoral hernias, it is probably not advisable in this setting. An open approach is probably preferred and can be a tissue- to-tissue repair. Any attempt at reduction is probably contraindi- cated because of the high potential for strangulation. If the bowel drops into the abdomen during an open approach, insertion of a scope into the hernia sac may be useful in evaluating the integrity of the affected bowel.

114
Q

A 55-year-old man with liver failure and ascites has an enlarging umbilical hernia. He has never undergone diuretic therapy. The correct therapy is:

A. Open repair with a waterproof mesh

B. High-volume paracentesis immediately before repair

C. Deferring hernia repair until correction of the ascites by maximal medical therapy, transjugular intrahepatic portosystemic shunting, or liver transplantation

D. Laparoscopic repair with an inlay mesh

E. Repair of the hernia and use of an abdominal binder after the operation

A

C

Repair of any hernia in a patient with ascites is a challenging problem. In general, any consideration of an elective repair should be deferred until the ascites is controlled with maximal medical therapy.

If the ascites can be controlled, umbilical hernias should be repaired in patients with cirrhosis because of the high morbidity and mortality if the hernia ruptures.

If there is skin breakdown and leakage of ascites, an urgent repair may be necessary to prevent peritonitis.

Frequent paracentesis may be helpful in this difficult scenario with associated high morbidity.

115
Q

Match the following Surgical Wound Classification with the hernia repair scenario:

A. Sterile

B. Clean

C. Clean contaminated

D. Contaminated

E. Dirty

I: Explantation of infected mesh

II: Symptomatic reducible inguinal hernia

III: Strangulated inguinal hernia with ischemic bowel

IV: Not a surgical wound class

V: Small bowel enterotomy during ventral hernia repair

A

A-IV; B-II; C-V; D-III; E-I

A clean wound is only in contact with the normal skin flora and should have a surgical site infection rate of <2%. An elective inguinal hernia repair is a clean wound.

A clean-contaminated wound enters a viscus in a controlled manner; the hernia has a surgical site infection risk of 2%–10%. Gynecologic cases, prepped colon or small bowel procedures, and head and neck cases are clean-contaminated cases.

A contaminated wound has intestinal spillage without active signs of infection and has an infection rate of ∼10% even with antibiotics. Examples of a contaminated wound are ischemic bowel and an unprepped colon.

Dirty cases are surgeries with active infection such as an empyema and perforated appendicitis.

During a hernia repair, the wound class is very important to help guide mesh placement.

A synthetic mesh can be used with clean and clean-contaminated wounds.

Biologic meshes are used for contaminated or dirty wounds if a mesh is needed.

“Sterile” is not a part of the Surgical Wound Classification.

116
Q

Which of the following statements is true regarding femoral hernias?

A. Femoral hernias should not be repaired through an infrainguinal approach.

B. If an incarcerated femoral hernia cannot be reduced intraoperatively, the insertion of the inguinal ligament can be cut from the pubic tubercle to allow more space.

C. Femoral hernias are more common than inguinal hernias in females.

D. A laparoscopic repair is an inappropriate choice for a femoral hernia repair.

E. Incarcerated femoral hernias can be observed and managed on an elective basis.

A

B

Although femoral hernias are found more often in females than in males, inguinal hernias are still more common than femoral hernias.

Femoral hernias with small orifices in women are repaired from below the inguinal ligament with a few sutures or plugged with a cone of polypropylene mesh because they are rarely associated with hernias above the inguinal ligament.

Large femoral hernias can be repaired with the McVay Cooper’s ligament procedure or even better with a preperitoneal permanent prosthesis placed either laparoscopically or using an open preperitoneal approach.

Femoral hernias should be repaired promptly because the risk of incarceration is high.

Viability of the intestine must be ensured since incarceration of the antimesenteric border of the intestine (Richter hernia) could result in infarction. The presence of bloody fluid in an otherwise empty sac mandates a careful examination of the intestine to rule out ischemia.

The McVay repair may be preferred with incarcerated femoral hernias to avoid infection of the prosthetic mesh.

If the hernia sac is too large to reduce through the hernia defect, the medial insertion of the inguinal ligament can be cut from the pubic tubercle to allow more space. After reduction, the inguinal ligament is reattached to the pubic tubercle or Cooper’s ligament.

117
Q

Correct statements regarding the management of an incarcerated groin hernia include all of the following except:

A. A giant inguinal hernia is a chronically incarcerated hernia containing the majority of the bowel. After reduction, loss of abdominal domain and elevated intraabdominal pressures can be a concern.

B. Evaluation of the contents of the hernia sac is a step required in the repair of an incarcerated hernia.

C. Incarcerated femoral hernias can be repaired on an elective basis.

D. A hydrocele may mimic an incarcerated hernia.

E. Inadvertent reduction of incarcerated hernia contents during the induction of anesthesia does not ensure bowel viability.

A

C

Incarceration with potential resultant strangulation of small bowel is a serious complication of groin hernias. If the patient has an incarcerated inguinal hernia and strangulation is not suspected, an attempt at reduction by using sedation, Trendelenburg positioning, and gentle sustained pressure over the groin mass is appropriate.

Reduction en masse refers to the persistent nature of incarcerated tissue frequently through the external ring despite an apparently successful reduction.

If there is any indication of strangulation, reduction should not be attempted preoperatively. Rather, the sac should first be opened before reduction to inspect the viability of the contents. The presence of bloody fluid in the peritoneal cavity should raise the question of intestinal viability.

A delayed repair following a successful reduction may permit resolution of edema.

A hydrocele can mimic an incarcerated hernia. Should physical examination fail to establish the diagnosis, a hydrocele will transilluminate clearly but a hernia will not. Ultrasound can confirm the presence of a hydrocele.

In patients with a suspected strangulated hernia, spontaneous reduction of the hernia’s contents could occur. However, the surgeon should not assume that the bowel is viable, and examination of the abdominal contents is mandatory.

An incarcerated femoral hernia should always be repaired on an urgent basis, as the rate of strangulation is high.

Giant inguinal hernias are often chronically incarcerated and can contain the majority of the intraabdominal contents. After reduction, the volume of the abdominal contents is increased enough that the abdominal wall closure can become problematic.

118
Q

A 55-year-old obese woman returns to the office 2 weeks after an open incisional hernia repair with mesh complaining of increasing pain and redness. On examination, the wound is red and fluctuant consistent with a postoperative wound infection. Which of the following is true regarding wound infections?

A. Laparoscopic hernia repairs have lower rates of wound infections than open repairs.

B. Polytetrafluoroethylene (PTFE) mesh is an absorbable mesh that can be left in place even with a wound infection.

C. Suture ventral hernia repair has higher infection rates and recurrence rates than mesh ventral hernia repair.

D. Surgical site infections have no effect on the recurrence rate after a ventral hernia repair.

E. Closed and open repairs have high rates of systemic infections.

A

A

The advantage of mesh in a hernia repair is that it decreases hernia recurrence from 8.2% to 2.7%, but has slightly high rates of infection. Mesh infection rates are 3%–10% after a hernia repair. Meta-analysis shows that a laparoscopic ventral hernia repair with a mesh has infection rates of 1%–2% compared with ∼10% for an open ventral hernia repair with mesh, with no difference in the recurrence rate.

PTFE is a synthetic mesh that becomes encapsulated with no ingrowth of tissue; if infected, it must be explanted.

Macroporous monofilament meshes are more resistant to infection because macrophages can enter the pores and combat infection.

Surgical site infection is the most common reason that a mesh is explanted and leads to higher recurrence rates.

119
Q

Which of the following developments has not led to a decrease in recurrence rates after a groin hernia repair?

A. Modifications of the Bassini repair

B. Routine use of prosthetic material

C. Widespread acceptance of the “tension-free” concept

D. Use of the preperitoneal space for hernia repair

E. Use of laparoscopy in hernia repair

A

ANSWER: A COMMENTS: Recurrence rates for groin hernias vary from less than 1% to 30%. True recurrence rates are difficult to establish because of inadequate patient follow-up. The Bassini repair and its modifications (Shouldice and McVay) all create tension at the suture line and have been found to have recurrence rates between 10% and 30% when performed outside specialized centers. Several developments in the latter half of the 20th century have significantly influenced the currently accepted level of a recurrence rate of less than 5%. The routine uses of prosthetic material to perform a tension-free hernia repair became accepted by surgeons after being popularized by Lichtenstein in the 1980s. Others, such as Rutkow, Robbins, Kugel, Gilbert, Wantz, Stoppa, Millikan, and Nyhus, have used multiple prosthetic materials and approaches to continue to reduce the recurrence rate to below 1%. The most popular prosthetic materials are polypropylene, polyester fiber mesh, and PTFE. Use of the preperitoneal space also helped lower recurrence rates by allowing larger pieces of prosthetic material to be used and incorporating intraabdominal pressure to aid in keeping the mesh in place. Laparoscopy by itself has not helped lower recurrence rates below those achieved with open tension-free mesh repairs, but it has given the surgeon another option for accessing the preperitoneal space.

120
Q

Which of the following is true regarding characteristics of mesh used in hernia repair?

A. Lightweight mesh, compared with heavyweight mesh, provokes an intense inflammatory reaction in the body causing scarring, pain, and shrinkage.

B. Synthetic meshes can never be placed into a contami- nated field.

C. Polypropylene meshes have high adhesion risks, so they should not be placed directly next to bowel.

D. Biologic meshes are processed human, bovine, and porcine tissues composed of a collagen matrix and immune cells.

E. Use of a mesh reduces the rate of surgical site infections.

A

C

Synthetic meshes are composed of PTFE, polypropylene, or polyester. Porosity is important in determining the flexibility and infection resistance of the mesh. Large pores decrease the inflammatory foreign body reaction and allow improved penetration by immune cells.

Lightweight meshes have large pores with thinner filaments, and heavyweight meshes have small pores with thicker filaments leading to a denser material.

Heavyweight meshes are less elastic, which can interfere with the abdominal wall compliance, and have more shrinkage than lightweight meshes.

Polypropylene meshes have high rates of adhesion and cannot come into contact with bowel, and PTFE has less adhesion risk.

Often composite materials are used with a polypropylene side to promote adherence to the abdominal wall and a PTFE side against the viscera.

Biologic meshes are composed of acellular collagen matrices that have been derived from either porcine small intestine or human dermis.

Using mesh reduces the rate of recurrence but increases the rate of surgical site infections. Traditional teaching was that in a contaminated field (i.e., with a bowel resection or enterotomy), a synthetic mesh could never be used, but newer data show acceptable rates of surgical site infections in select circumstances.

121
Q

All of the following statements concerning the Lichtenstein repair are true except:

A. It is performed with local anesthesia in an outpatient setting.

B. Polypropylene is the most common prosthetic material used for the repair.

C. The medial edge of the mesh is sutured to the transversalis fascia, and the lateral edge is sutured to the inguinal ligament.

D. To reduce recurrence rates, the most cephalad tails of the mesh should extend 2 to 4 cm beyond the internal ring.

E. To reduce recurrence rates, the most caudal aspect of the mesh should extend at least 2 cm over the pubic tubercle.

A

C

As commonly performed, the open herniorrhaphy technique is a tension-free repair popularized by Irving L. Lichtenstein and colleagues.

The Lichtenstein repair is routinely performed in an outpatient setting with local anesthesia. A polypropylene mesh is most commonly sutured medially to the transversus abdominis arch, with the internal oblique being overlapped by approximately 2 cm. The latter edge of the mesh is sutured to the inguinal ligament.

To reduce recurrence rates, Parviz Amid has described overlapping the mesh at least 2 cm over the pubic tubercle and 2 to 4 cm lateral to the internal ring.

The Lichtenstein repair was one of the first prosthetic repairs to achieve approximately an overall 1% or lower recurrence rate in the United States.

122
Q

A morbidly obese man is in the ICU 12 h after a separation- of-components hernia repair for a large hernia with loss of abdominal domain. He is intubated and sedated. The nurse calls you to the bedside because of worsening abdominal distension. Which of the following is not a sign of developing an abdominal compartment syndrome?

A. Increasing peak airway pressures

B. Decreasing urine output

C. Increasing tidal volumes

D. Increasing bladder pressures

E. Decreased central venous pressure (CVP)

A

C

Abdominal compartment syndrome is a serious complication that can develop after a separation-of-component hernia repair, especially if there was a large loss of abdominal domain preoperatively.

Signs of increasing abdominal hypertension include elevated peak airway pressures, declining tidal volumes, increasing abdominal distension and firmness, decreasing urine output, and increasing bladder pressures.

Intraabdominal hypertension is defined as sustained pressure > 12 mmHg, and abdominal compartment syndrome is sustained pressures > 20 mmHg with evidence of end-organ dysfunction.

Since the measurement of intraabdominal pressure can be influenced by many factors, these numbers are taken within a clinical context. The concern is that with increasing abdominal pressure, intraabdominal organs will become ischemic, and it will be difficult to ventilate.

The steps to reduce pressure and improve perfusion include deepening sedation, paralysis, and, as a last resort, reopening the abdomen.

123
Q

Which of the following is not true regarding laparoscopic hernia repair?

A. Local anesthesia with sedation is the most common form of anesthesia used.

B. It could lead to injury to the genitofemoral nerve and the lateral femoral cutaneous nerve.

C. Transabdominal preperitoneal or total extraperitoneal approaches are commonly used.

D. Fixation devices for the mesh should not be placed below the iliopubic tract.

E. It is best suited for recurrent and bilateral hernias.

A

A

Laparoscopic techniques for the repair of inguinal hernias were introduced in the 1990s and have gained mild-to- moderate acceptance, with less than 10% of all inguinal hernia repairs being performed via these approaches.

The repairs are usually performed with the patient under general anesthesia, and the cost is considerably higher than that of an open approach with local anesthesia and sedation.

Although there is controversy regarding its use for unilateral, newly diagnosed hernias, it seems ideally suited for recurrent and bilateral hernias, where the disability and technical difficulty associated with open (conventional) repairs cannot be overlooked.

Laparoscopy can be performed totally extraperitoneally by dissecting within the preperitoneal space or transabdominally. In either case, a preperitoneal repair is performed.

Mesh fixation devices placed below the iliopubic tract risk injury to the genitofemoral nerve and the lateral femoral cutaneous nerve. Placement of fixation devices is also avoided below the internal inguinal ring in an area known as the “triangle of doom.” This triangle is bordered laterally by the spermatic vessels and medially by the vas deferens. Located within this triangle are the external iliac artery and vein and the femoral nerve.

124
Q

A 65-year-old man underwent sigmoid resection and end colostomy for perforated diverticulitis. One year after the surgery he presents to your office complaining of a bulge around his stoma and an inability to obtain a good seal on his appliance. Which of the follow is true regarding parastomal hernias?

A. Ileostomies develop parastomal hernias more often than colostomies.

B. Parastomal hernias are best prevented by making the fascial defect as small as possible during the initial operation and placing the intestine through the rectus sheath.

C. The only operative repair option is to relocate the stoma.

D. Parastomal hernias have large enough fascial defects that intestinal strangulation does not happen.

E. Every parastomal hernia should be repaired emergently to prevent high rates of strangulation.

A

B

Parastomal hernias are a very common problem after stoma formation, affecting up to 30% of stomas. Colostomies are more likely to develop hernias than ileostomies.

Hernias can be asymptomatic, painful, cause skin irritation from poorly fitting appliances, or lead to intestinal strangulation. Only symptomatic parastomal hernias should be repaired.

Stomas should always be formed through the rectus sheath with the smallest defect necessary, and placement of a mesh at initial creation is not routine.

There are three main methods to repair a parastomal hernia: relocation, keyhole mesh (either synthetic or biologic), or Sugarbaker repair—a single mesh is placed underneath the defect and the intestine is lateralized.

125
Q

Which of the following urologic complications does not occur with hernia repair?

A. Ischemic orchitis

B. Transection of the vas deferens

C. Prostatitis

D. Testicular atrophy

E. Ovarian torsion within the hernia sac

A

C

Transection of the vas deferens if identified intra-operatively should be repaired with interrupted fine monofilament suture. A crush injury and stretch can damage the vas as well without complete transection.

There appears to be a small association between men who had a hernia repair as a child and adult infertility, but the exact mechanism is not defined.

Testicular atrophy develops from ischemia caused by an incarcerated hernia or intraoperative injury. It occurs at a rate of 2.6%–5%, but testicles should only be removed for frank necrosis on intraoperative examination.

Ischemic orchitis presents as postoperative testicular pain caused by venous thrombosis from the overaggressive handling of the pampiniform plexus. The occurrence is 1% and is managed conservatively with nonsteroidal antiinflammatory drugs (NSAIDs).

Ovarian strangulation can be as high as 30% if found within an incarcerated hernia, primarily from ovarian torsion.

126
Q

There are many different techniques to repair a ventral hernia. In the last few years, a new technique, the transversus abdominis muscle release (TAR), has been developed to repair large ventral hernias with the loss of domain. Which of the following is false regarding TAR?

A. TAR is a modification of the Rives-Stoppa retromuscular hernia repair.

B. TAR has higher wound complication rates than a traditional separation-of-components hernia repair.

C. To gain more abdominal wall length, the posterior rectus sheath is incised 0.5 to 1 cm medial to the linea semilu- naris and the transversus abdominal muscle attachments are cut.

D. The transversus abdominis is mobilized from the costal margin to the space of Retzius.

E. A mesh can be placed in the retrorectus space to reinforce the hernia repair.

A

B

A TAR is a newer modification of the Rives-Stoppa retromuscular hernia repair. Described by Rosen et al., the repair utilizes the retromuscular space.

The posterior rectus sheath is entered and the posterior fascia dissected off the muscle laterally. At 0.5 to 1 cm medial to the linea semilunaris, the posterior rectus sheath is incised and the transversus abdominis muscle attachment is cut. The transversus abdominis is cut cranially to the costal margin and caudally into the space of Retzius.

Releasing the transversus abdominis creates laxity of the abdominal wall allowing the posterior rectus fascia to be approximated along the midline. A large sublay mesh is placed into the retrorectus space and tacked in place with a few interrupted sutures. Drains are placed, the anterior rectus sheath is approximated, and the skin closed.

In comparison to the traditional separation-of-components technique, additional abdominal wall domain is gained without raising large subcutaneous flaps so the blood supply to the abdominal wall is preserved. This avoids the wound complications usually associated with a separation-of-components repair. In the initial study, the long-term recurrence rate was ∼5%.

127
Q

Which of the following statements is not true with regard to incisional ventral hernias?

A. Primary repairs are associated with a 30%–50% recurrence rate.

B. The incidence of incisional hernias is between 2% and 11% after laparotomy.

C. Prosthetic mesh repairs have reduced the recurrence rate to 20% or less.

D. Bilayer mesh can be placed safely in the intraabdominal cavity.

E. Comorbid conditions, such as diabetes, hypertension, and obesity, are uncommon in patients with incisional hernias.

A

E

In the United States, approximately 2 million laparotomies are performed each year, with a reported incisional ventral hernia rate between 2% and 11%. The population of patients in whom wound dehiscence occurs tends to be obese, and they frequently might have one or more of the following: comorbidities of a smoking history, hypertension, and diabetes.

Primary incisional ventral hernia repairs have been associated with recurrence rates of up to 50%. Prosthetic mesh repairs have lowered the recurrence rates to less than 10%.

Recently, it has been found that a bilayer prosthesis composed of both polypropylene and PTFE can be placed safely in the abdominal cavity without the development of bowel obstruction or enterocutaneous fistulas.

Intraabdominal placement of the mesh allows the greatest underlay of the fascial defect, thereby enabling the greatest amount of tissue ingrowth to occur. When polypropylene alone is placed in the intraabdominal cavity, bowel obstruction, enterocutaneous fistula, and difficult reentry to the abdomen occur with an unacceptable frequency.

There is increasing evidence of less recurrence with a laparoscopic incisional hernia repair using a mesh. This is typically an onlay with 3 to 5 cm overlap of the fascial defect. Local wound problems seem to be decreased.

Lighter synthetic mesh is gaining in popularity. Again, expertise and experience in this advanced laparoscopic procedure are necessary to achieve low recurrence rates and avoid serious complications.

A frequent but easily treated complication of a laparoscopic repair is seroma, which can occur in up to 30%–50% of patients. It is usually self-limited. The use of drains to avoid seroma formation is controversial, and it has not been associated with a decrease in its incidence. Aspiration of seromas is best accomplished under image guidance if they are symptomatic or concern about infection exists.

128
Q

A 55-year-old man who runs marathons has a recurrent inguinal hernia. Which statement is correct?

A. The previous type of repair has no significance in the treatment plan.

B. A Shouldice repair is recommended.

C. He will have to stop running marathons after repair.

D. A repair can be performed with the patient under local anesthesia with a high likelihood of success.

E. Laparoscopic repair, if the previous repair was performed in an open manner with a mesh, is an evidence-based choice.

A

E

The repair of a recurrent inguinal hernia can be challenging in terms of preventing recurrence and avoiding morbidity such as chronic pain.

Tissue-to-tissue repairs may be repaired with an anterior mesh repair such as the Lichtenstein or plug-and-patch repair.

Obtaining a prior operative report is strongly recommended to facilitate selection of the appropriate current repair.

A laparoscopic repair for recurrence of a hernia after an open anterior mesh repair is supported by prospective trials. The caveat is that such results require expertise and experience in laparoscopic repair.

A Shouldice repair is not generally recommended for recurrent hernias.

A repair by an expert surgeon should allow resumption of all normal activity and is recommended for active individuals.

A repair can be deferred if asymptomatic in selected patients.

129
Q

A 30-year-old man presents to your office complaining of groin pain while playing sports. On examination, he has no palpable bulge but does experience tenderness with palpation over the pubic tubercle. Which of the following is true regarding a sports hernia or athletic pubalgia?

A. Athletic pubalgia is most common in long-distance runners.

B. Even though a hernia defect is not palpable on examination, it is clearly apparent on imaging.

C. There is no surgical role for the management of athletic pubalgia.

D. Operative management can be performed with open or laparoscopic hernia repair technique.

E. Athletic pubalgia is an acute traumatic tear of the inguinal ligament.

A

D

The component of groin pain or a sports hernia treated by general surgeons is more precisely referred to as athletic pubalgia. This is a tear of the intersection of the rectus abdominis or adductor longus to the pubis from repetitive stress.

The typical populations with athletic pubalgia are participants in sports with high-speed changes in direction and twisting such as hockey, soccer, wrestling, and football. They complain of pain with motion—especially hip adduction—but there is no visible bulge as there is no defect in the abdominal wall.
On examination, there is point tenderness over the pubic tubercle.

The inflammation and tear at the pubis can be identified on magnetic resonance imaging (MRI). Dynamic ultrasound shows laxity and bulge of the rectus muscle with no distinct hernia.

Nonoperative management includes rest, physical therapy, and steroid injections.

If nonoperative management fails, then an inguinal hernia repair (open or laparoscopic) can be used to resolve the pain.

A hernia repair is thought to support the laxity of the pelvic floor and cause an inflammatory reaction reattaching the torn aponeurosis to the pubic tubercle. Some surgeons perform a nerve transection of the iliohypogastric and ilio-inguinal nerves.

130
Q

The separation-of-components technique:

A. Is best for hernias with fascial defects of 3 cm or less

B. Has a recurrence rate of approximately 10%

C. May be used when there is contamination or bowel surgery is required

D. Is contraindicated for recurrent incisional hernias

E. Ideally avoids the use of a mesh

A

ANSWER: C COMMENTS: Although a primary suture repair with mesh has acceptable recurrence rates for small incisional hernias, recurrence rates are disappointing when these techniques are used for very large hernias. In addition, more complications are associated with a mesh repair of very large hernias either by open or by laparoscopic technique. The separation-of-components technique has demonstrated improved results in the repair of massive incisional hernias, with recurrence rates of approximately 20%. This technique can be used when there is contamination or bowel surgery is required, thereby avoiding the dreaded complication of mesh infection. The separation-of-components technique can be used for failed mesh repairs. In selected cases, the addition of soft synthetic mesh has improved success with the technique.

131
Q

The incidence of inguinal hernias in men has a bimodal distribution, which peaks A. Before the second year of life and after age 50. B. Before the first year of life and after age 40. C. Before the eighth year of life and after age 40. D. Before the fifth year of life and after age 50.

A

Answer: B Approximately 75% of abdominal wall hernias occur in the groin. The lifetime risk of inguinal hernia is 27% in men and 3% in women. Of inguinal hernia repairs, 90% are performed in men and 10% in women. The incidence of inguinal hernias in men has a bimodal distribution, with peaks before the first year of life and after age 40. Abramson demonstrated the age dependence of inguinal hernias in 1978. Those between ages 25 and 34 years had a lifetime prevalence rate of 15%, whereas those aged 75 years and over had a rate of 47% (Table 37-1). Approximately 70% of femoral hernia repairs are performed in women; however, inguinal hernias are five times more common than femoral hernias. The most common subtype of groin hernia in men and women is the indirect inguinal hernia. (See Schwartz 10th ed., p. 1495.)

132
Q

The two types of collagen found to exist in a decreased ratio of the skin of inguinal hernia patients are A. Types I and II B. Types II and III C. Types I and III D. Types III and VI

A

Answer: C Epidemiologic studies have identified risk factors that may predispose to a hernia. Microscopic examination of skin of inguinal hernia patients demonstrated significantly decreased ratios of type I to type III collagen. Type III collagen does not contribute to wound tensile strength as significantly as type I collagen. Additional analyses revealed disaggregated collagen tracts with decreased collagen fiber density in hernia patients’ skin. Collagen disorders, such as Ehlers-Danlos syndrome, are also associated with an increased incidence of hernia formation ( able 37-2). Recent studies have found an association between concentrations of extracellular matrix elements and hernia formation. Although a significant amount of work remains to elucidate the biologic nature of hernias, current evidence suggests they have a multifactorial etiology with both environmental and hereditary influences. (See Schwartz 10th ed., able 37-4, p. 1502.)

133
Q

According to the Nyhus classification system that categorizes hernia defects by location, size, and type, type IIIC represents A. Indirect hernia; internal abdominal ring normal; typically in infants, children, small adults B. Direct hernia; size is not taken into account C. Recurrent hernia; modifiers A-D are sometimes added, which correspond to indirect, direct, femoral, and mixed. D. Femoral hernia

A

Type I Indirect hernia; internal abdominal ring normal; typically in infants, children, small adults Type II Indirect hernia; internal ring enlarged without impingement on the floor of the inguinal canal; does not extend to the scrotum Type IIIA Direct hernia; size is not taken into account Type IIIB Indirect hernia that has enlarged enough to encroach upon the posterior inguinal wall; indirect sliding or scrotal hernias are usually placed in this category because they are commonly associated with extension to the direct space; also includes pantaloon hernias Type IIIC Femoral hernia Type IV Recurrent hernia; modifiers A–D are sometimes added, which correspond to indirect, direct, femoral, and mixed, respectively

134
Q

Taxis A. Should be performed when strangulation is suspected. B. Refers to the method of securing mesh to the inguinal ligament. C. Should not be repeated more than five times. D. Should be used for incarcerated hernias without sequelae or strangulation.

A

Answer: D Incarceration occurs when hernia contents ail to reduce; however, a minimally symptomatic, chronically incarcerated hernia may also be treated nonoperatively. Taxis should be attempted for incarcerated hernias without sequelae of strangulation, and the option of surgical repair should be discussed prior to the maneuver. To perform taxis, analgesics and light sedatives are administered, and the patient is placed in the Trendelenburg position. The hernia sac is elongated with both hands, and the contents are compressed in a milking fashion to ease their reduction into the abdomen. The indication or emergent inguinal hernia repair is impending compromise of intestinal contents. As such, strangulation of hernia contents is a surgical emergency. Clinical signs that indicate strangulation include fever, leukocytosis, and hemodynamic instability. The hernia bulge is usually warm and tender, and the overlying skin may be erythematous or discolored. Symptoms of bowel obstruction in patients with sliding or incarcerated inguinal hernias may also indicate strangulation. Taxis should not be performed when strangulation is suspected, as reduction of potentially gangrenous tissue into the abdomen may result in an intra-abdominal catastrophe. Preoperatively, the patient should receive fluid resuscitation, nasogastric decompression, and prophylactic intravenous antibiotics. (See Schwartz 10th ed., p. 1505.)

135
Q

A hernia sac that extends into the scrotum may A. Require extensive dissection and reduction B. Require division within the inguinal canal C. Require amputation of the sac D. Require the sac to be inverted into the preperitoneum

A

Answer: B In cases where the viability of sac contents is in question, the sac should be incised, and hernia contents should be evaluated for signs of ischemia. The defect should be enlarged to augment blood low to the sac contents. Viable contents may be reduced into the peritoneal cavity, while nonviable contents should be resected, and synthetic prostheses should be avoided in the repair. In elective cases, the sac may be amputated at the internal inguinal ring or inverted into the pre-peritoneum. Both methods are effective; however, patients undergoing sac excision had significantly increased postoperative pain in a prospective trial. Dissection of a densely adherent sac may result in injury to cord structures and should be avoided; however, sac ligation at the internal inguinal ring is necessary in these cases. A hernia sac that extends into the scrotum may require division within the inguinal canal, as extensive dissection and reduction risks injury to the pampiniform plexus, resulting in testicular atrophy and orchitis. At this point, the inguinal canal is reconstructed, either with native tissue or with prostheses. The following sections describe the most commonly performed types of tissue-based and prosthetic-based reconstructions. (See Schwartz 10th ed., p. 1505.)

136
Q

The technique indicated for femoral hernias in cases where prosthetic material is contraindicated is A. The Bassini repair B. The Shouldice repair C. The McVay repair D. Lichtenstein tension-free repair

A

Answer: C The McVay repair addresses both inguinal and femoral ring defects. This technique is indicated for femoral hernias and in cases where the use of prosthetic material is contraindicated (Fig. 37-1). Once the spermatic cord has been isolated, an incision in the transversalis fascia permits entry into the pre- peritoneal space. The upper lap is mobilized by gentle blunt dissection of underlying tissue. Cooper ligament is bluntly dissected to expose its surface. A 2- to 4-cm relaxing incision is made in the anterior rectus sheath vertically from the pubic tubercle. This incision is essential to reduce tension on the repair; however, it may result in increased postoperative pain and higher risk of ventral abdominal herniation. Using either interrupted or continuous suture, the superior transversalis flap is then fastened to Cooper ligament, and the repair is continued laterally along Cooper ligament to occlude the femoral ring. Lateral to the femoral ring, a transition stitch is placed, affixing the transversalis fascia to the inguinal ligament. The transversalis is then sutured to the inguinal ligament laterally to the internal ring. (See Schwartz 10th ed., Figure 37-17, pp. 1507–1508.)

137
Q

General anesthesia induction resulting in reduction of an incarcerated or strangulated inguinal hernia during laparoscopic repair A. Requires no action B. Requires abdominal exploration or nonviable tissue C. Requires immediate conversion to an open repair D. Requires proceeding with a totally extraperitoneal repair

A

Answer: B Laparoscopic inguinal hernia repairs reinforce the abdominal wall via a posterior approach. Principal laparoscopic methods include the transabdominal preperitoneal (TAPP) repair, the totally extraperitoneal (TEP) repair, and the less commonly performed intraperitoneal onlay mesh (IPOM) repair. Although laparoscopic repairs in experienced hands are relatively expedient, they necessitate the administration of general anesthesia and its inherent risks. Any patient with a contraindication to the use of general anesthesia should not undergo laparoscopic hernia repair. Occasionally, general anesthesia induction may result in reduction of an incarcerated or strangulated inguinal hernia. If the surgeon suspects this might have occurred, the abdomen should be explored or nonviable tissue either via laparoscopy or upon conversion to an open laparotomy. (See Schwartz 10th ed., p. 1509.)

138
Q

The medical issue NOT associated with hernia recurrence A. Malnutrition B. Steroid use C. Smoking D. Alcohol use

A

Answer: D When a patient develops pain, bulging, or a mass at the site of an inguinal hernia repair, clinical entities such as seroma, persistent cord lipoma, and hernia recurrence should be considered. Common medical issues associated with recurrence include malnutrition, immunosuppression, diabetes, steroid use, and smoking. Technical causes of recurrence include improper mesh size, tissue ischemia, infection, and tension in the reconstruction. A focused physical examination should be performed. As with primary hernias, ultrasound, (CT), or magnetic resonance imaging (MRI) can elucidate ambiguous physical findings. When a recurrent hernia is discovered and warrants reoperation, an approach through a virgin plane facilitates its dissection and exposure. Extensive dissection of the scarred field and mesh may result in injury to cord structures, viscera, large blood vessels, and nerves. After an initial anterior approach, the posterior laparoscopic approach will usually be easier and more effective than another anterior dissection. Conversely, failed preperitoneal repairs should be approached using an open anterior repair. (See Schwartz 10th ed., p. 1514.)

139
Q

Nociceptive pain is A. The result of ligamentous or muscular trauma and inflammation. B. The result of direct nerve damage or entrapment. C. The result of pain conveyed through afferent autonomic pain fibers. D. Characterized as acute.

A

Answer: A Pain after inguinal hernia repair is classified into acute or chronic manifestations of three mechanisms: nociceptive (somatic), neuropathic, and visceral pain. Nociceptive pain is the most common of the three. Because it is usually a result of ligamentous or muscular trauma and inflammation, nociceptive pain is reproduced with abdominal muscle contraction Treatment consists of rest, nonsteroidal anti-inflammatory drugs (NSAIDs), and reassurance, as it resolves spontaneously in most cases. Neuropathic pain occurs as a result of direct nerve damage or entrapment. It may present early or late, and it manifests as a localized, sharp, burning, or tearing sensation. It may respond to pharmacologic therapy and to local steroid or anesthetic injections when indicated. Visceral pain refers to pain conveyed through afferent autonomic pain fibers. It is usually poorly localized and may occur during ejacula- tion as a result of sympathetic plexus injury. (See Schwartz 10th ed., p. 1514.)

140
Q

The triangle of pain is bordered by all of the following EXCEPT A. Iliopubic tract B. Ductus deferens C. Gonadal vessels D. Reflected peritoneum

A

Answer: B The preperitoneal anatomy seen in laparoscopic hernia repair led to characterization of important anatomic areas of interest, known as the triangle of doom, the triangle of pain, and the circle of death (Fig. 37-2). The triangle of doom is bordered medially by the vas deferens and laterally by the vessels of the spermatic cord. The contents of the space include the external iliac vessels, deep circumflex iliac vein, femoral nerve, and genital branch of the genitofemoral nerve. The triangle of pain is a region bordered by the iliopubic tract and gonadal vessels, and it encompasses the lateral femoral cutaneous, femoral branch of the genitofemoral, and femoral nerves. The circle of death is a vascular continuation formed by the common iliac, internal iliac, obturator, inferior epigastric, and external iliac vessels. (See Schwartz 10th ed., able 37-9, pp. 1499 and 1501.)

141
Q

The most common cause of urinary retention after hernia repair is A. General anesthesia B. Narcotic analgesia C. Pain D. Perioperative bladder distention

A

Answer: A The most common cause of urinary retention after hernia repair is general anesthesia, which is routine in laparoscopic hernia repairs. Among 880 patients undergoing inguinal hernia repair with local anesthesia only 0.2% developed urinary retention, whereas the rate of urinary retention was 13% among 200 patients undergoing repair with general or spinal anesthesia. Other risk factors for postoperative urinary retention include pain, narcotic analgesia, and perioperative bladder distention. Initial treatment of urinary retention requires decompression of the bladder with short-term catheteriza- tion. Patients will generally require an overnight admission and trial of normal voiding before discharge. Failure to void normally requires reinsertion of the catheter for up to a week. Chronic requirement of a urinary catheter is rare, although older patients may require prolonged catheterization. (See Schwartz 10th ed., p. 1515.)

142
Q

The outcome found more commonly with TAPP compared with TEP repair is A. Length of stay B. Time to recovery C. Risk of intra-abdominal injuries D. Higher short-term recurrence rates

A

Answer: C Although controversy persists regarding the utility of TEP versus TAPP, reviews to date find no significant differences in operative duration, length of stay, time to recovery, or short-term recurrence rate between the two approaches. In TAPP repair, the risk of intra-abdominal injury is higher than in TEP repair. This finding prompted the International Endohernia Society (IEHS) to recommend that TAPP should only be attempted by surgeons with sufficient experience. A Cochrane systematic review found that rates of port-site hernias and visceral injuries were higher for the TAPP technique, whereas TEP may be associated with a higher rate of conversion to an alternative approach; however, neither finding was sufficiently compelling to recommend one technique over the other. (See Schwartz 10th ed., p. 1517.)

143
Q

The ratio of inguinal hernias to femoral hernias is: A. 7:3 B. 5:1 C. 8:2 D. 10:1

A

Answer: B Approximately 75% of abdominal wall hernias occur in the groin. The lifetime risk of inguinal hernia is 27% in men and 3% in women. Of inguinal hernia repairs, 90% are performed in men and 10% in women. The incidence of inguinal hernias in men has a bimodal distribution, with peaks before the first year of age and after age 40. Abramson demonstrated the age dependence of inguinal hernias in 1978. Those between 25 and 34 years had a lifetime prevalence rate of 15%, whereas those aged 75 years and over had a rate of 47% ( able 37-1). Approximately 70% of femoral hernia repairs are performed in women; however, inguinal hernias are five times more common than femoral hernias. The most common subtype of groin hernia in men and women is the indirect inguinal hernia. (See Schwartz 10th ed., p. 1495.)

144
Q

The high incidence of inguinal hernias in preterm babies is most often due to A. Failure of the peritoneum to close B. Familial history C. Female gender D. Developmental dysplasia of the hip

A

Answer: A Inguinal hernias may be congenital or acquired. Most adult inguinal hernias are considered acquired defects in the abdominal wall although collagen studies have demonstrated a heritable predisposition. A number of studies have attempted to delineate the precise causes of inguinal hernia formation; however, the best-characterized risk factor is weakness in the abdominal wall musculature (Table37-4). Congenital hernias, which make up the majority of pediatric hernias, can be considered an impedance of normal development, rather than an acquired weakness. During the normal course of development, the testes descend from the intra-abdominal space into the scrotum in the third trimester. Their descent is preceded by the gubernaculum and a diverticulum of peritoneum, which protrudes through the inguinal canal and becomes the processus vaginalis. Between 36 and 40 weeks of gestation, the processus vaginalis closes and eliminates the peritoneal opening at the internal inguinal ring. Failure of the peritoneum to close results in a patent processus vaginalis (PPV), hence the high incidence of indirect inguinal hernias in preterm babies. Children with congenital indirect inguinal hernias will present with a PPV; however, a patent processus does not necessarily indicate an inguinal hernia (Fig.37-3). In a study of nearly 600 adults undergoing general laparoscopy, bilateral inspection revealed that 12% had PPV. None of these patients had clinically signiicant symptoms of a groin hernia. In a group of 300 patients undergoing unilateral laparoscopic inguinal hernia repair, 12% were found to have a contralateral PPV, which was associated with a fourfold 5-year incidence of inguinal hernia. (See Schwartz 10th ed., able 37-3 and Figure 37-10, pp. 1500 and 1502.)

145
Q

Injury to the lateral femoral cutaneous nerve results in A. Inguinodynia B. Osteitis pubis C. Meralgia paresthetica D. Nerve entrapment

A

Answer: C Other chronic pain syndromes include local nerve entrapment, meralgia paresthetica, and osteitis pubis. At greatest risk of entrapment are the ilioinguinal and iliohypogastric nerves in anterior repairs and the genitofemoral and lateral femoral cutaneous nerves in laparoscopic repairs. Clinical manifestations of nerve entrapment mimic acute neuropathic pain, and they occur with a dermatomal distribution. Injury to the lateral femoral cutaneous nerve results in meralgia paresthetica, a condition characterized by persistent paresthesias of the lateral thigh. Initial treatment of nerve entrapment consists of rest, ice, NSAIDs, physical therapy, and possible local corticosteroid and anesthetic injection. Osteitis pubis is characterized by inflammation of the pubic symphysis and usually presents as medial groin or symphyseal pain that is reproduced by thigh adduction. Avoiding the pubic periosteum when placing sutures and tacks reduces the risk of developing osteitis pubis. CT scan or MRI excludes hernia recurrence, and bone scan is confirmatory or the diagnosis. Initial treatment is identical to that of nerve entrapment; however, if pain remains intractable, orthopedic surgery consultation should be sought or possible bone resection and curettage. Irrespective of treatment, the condition often takes 6 months to resolve. (See Schwartz 10th ed., p. 1515.)

146
Q

The hernia repair method associated with the lowest recurrence rate is the A. Lichtenstein tension-free repair B. Open elective tissue-based repair C. Bassini technique D. Shouldice repair

A

Answer: A The incidence of recurrence is the most-cited measure of postoperative outcome following inguinal hernia repair. In evaluating the various available techniques, other salient signifiers of outcome include complication rates, operative duration, hospital stay, and quality of life. The following section summarizes the evidence-based outcomes of the various approaches to inguinal hernia repair. Among tissue repairs, the Shouldice operation is the most commonly performed technique, and it is most frequently executed at specialized centers. A 2012 meta-analysis from the Cochrane Database demonstrated significantly lower rates of hernia recurrence (odds ratio [OR] 0.62, confidence interval [CI] 0.45–0.85) in patients undergoing Shouldice operations when compared with other open tissue-based methods. In experienced hands, the overall recurrence rate or the Shouldice repair is about 1%. Although it is an elegant procedure, its meticulous nature requires significant technical expertise to achieve favorable outcomes, and it is associated with longer operative duration and longer hospital stay. One study found the recurrence rate for Shouldice repairs decreased from 9.4 to 2.5% after surgeons performed the repair six times. Compared with mesh repairs, the Shouldice technique resulted in significantly higher rates of recurrence (OR3.65,CI1.79–7.47); however, it is the most effective tissue-based repair when mesh is unavailable or contraindicated. Hernia recurrence is drastically reduced as a result of the Lichtenstein tension-free repair. Compared with open elective tissue-based repairs, mesh repair is associated with fewer recurrences (OR 0.37, CI 0.26–0.51) and with shorter hospital stay and faster return to usual activities. In a multi- institutional series, 3019 inguinal hernias were repaired using the Lichtenstein technique, with an overall recurrence rate of 0.2%. Among other tension-free repairs, the Lichtenstein technique remains the most commonly performed procedure worldwide. Meta-analysis demonstrates no significant differences in outcomes between the Lichtenstein and the plug and patch techniques; however, intra-abdominal plug migration and erosion into contiguous structures occurs in approximately 6% of cases. The Stoppa technique results in longer operative duration than the Lichtenstein technique. Nevertheless, postoperative acute pain, chronic pain, and recurrence rates are similar between the two methods. Perhaps the most compelling advantage of the Lichtenstein technique is that nonexpert surgeons rapidly achieve similar outcomes to their expert counterparts. Guidelines issued by the European Hernia Society recommend the Lichtenstein repair for adults with either unilateral or bilateral inguinal hernias as the preferred open technique. (See Schwartz 10th ed., p. 1516.)

147
Q

A sliding hernia A. Has an abnormally high recurrence rate after repair B. Can involve the bladder C. Is more common in the right groin D. Occurs almost exclusively in women

A

Answer: B Inguinal hernias may compress adjacent nerves, leading to generalized pressure, localized sharp pain, and refferred pain. Pressure or heaviness in the groin is a common complaint, especially at the conclusion of the day or following prolonged activity. Sharp pain tends to indicate an impinged nerve and may not be related to the extent of physical activity performed by the patient. Neurogenic pain may be referred to the scrotum, testicle, or inner thigh. Questions should be directed to elicit and characterize extrainguinal symptoms. A change in bowel habits or urinary symptoms may indicate a sliding hernia consisting of intestinal contents or involvement of the bladder within the hernia sac. (See Schwartz 10th ed., p. 1503.)

148
Q
  1. A 6-month-old boy presents with an inguinal hernia, first noticed 2 weeks after birth. What is the best treatment choice?

(A) Observation

(B) Laparotomy

(C) Surgical repair when the child is fully grown

(D) Surgical repair of the affected side

(E) Surgical repair of the affected side and exploration of the nonaffected side to search for and repair a sac that was not previously detected by clinical means

A
  1. (E) Inguinal hernias in infancy are almost always congenital and indirect and are often bilateral. Bilateral exploration is recommended, except when the surgery is performed for incarceration.
149
Q

A 60-year-old male presents with an inguinal hernia of recent onset. Which of the following statements are TRUE?

(A) The hernia is more likely to be direct than indirect.

(B) Presents through the posterior wall of the inguinal canal, lateral to the deep inguinal ring.

(C) Is covered anteriorly by the transversalis fascia.

(D) Is more likely than a femoral hernia to strangulate.

(E) The sac is congenital.

A

(A)

Hernias, which present in adult life are most often direct and aquired, rather than indirect.

They protrude through the transversalis fascia, which forms the medial half of the posterior wall of the inguinal canal and is located medial to the deep inguinal ring and deep epigastric vessels.

Strangulation of direct inguinal herniae is uncommon, probably because the neck of the sac tends to be wide, rather than narrow and constricting.

150
Q

A 70-year-old cigarette smoker presents with a right inguinal mass that has enlarged and has caused discomfort in recent months. He complains of recent difficulty with micturition and nocturia. The swelling, which does not extend to the scrotum, reduces when resting. What is the likely diagnosis?

(A) Direct inguinal hernia

(B) Strangulated indirect inguinal hernia

(C) Hydrocele

(D) Aneurysm of the femoral artery

(E) Cyst of the cord

A

(A)

Direct hernias are more common in older patients.

There is an increased incidence in patients with a chronic cough and prostatic obstruction.

They are rarely encountered in children and women.

This type of hernia does not extend to the scrotum and rarely undergoes strangulation.

151
Q

A 65-year-old female requires emergency surgery for a strangulated inguinal hernia. Which of the following is correct?

(A) The sac is formed by an unobliterated processus vaginalis.

(B) The hernia is direct rather than indirect.

(C) Such herniae never contain small intestine.

(D) Strangulation never results in bowel ischemia and gangrene requiring resection.

(E) Indirect inguinal herniae are never found in female patients.

A

(A)

Indirect inguinal hernia sacs are found less commonly in female patients.

They are formed by the unobliterated processus vaginalis of the peritoneum and allow for the entry of intraperitoneal viscera, such as loops of small intestine, omentum, and the likes.

Compromise of blood supply by constriction leads to strangulation.

152
Q

An otherwise healthy, 60-year-old male has been advised to undergo surgical treatment for a left ingunial hernia. Which of the following are acceptable standards of surgical treatment?

(A) Traditional surgical repair under general or local anesthesia

(B) Repair of the hernia and ipsilateral orchiectomy, in order to better assure closure of the inguinal canal and reduce the possibility of recurrence

(C) Laparotomy to perform a retroperitoneal repair

(D) Surgical exploration of the contralateral groin to search for an occult hernia sac and to remove it before a hernai develops

(E) The patient should be advised to wear a truss postoperatively, in order to reduce the incidence of recurrence

A

(A)

Traditional hernia repair is performed under local or general anesthesia. Laparoscopic repair, in general, is performed under general anesthesia.

Combinations of local anesthesia and intravenous sedation are in routine use.

Orchiectomy, though occasionally used in special circumstances, is by no means a routine part of hernia repair.

Exploration of the opposite groin, though recommended in infants, does not apply to adults.

153
Q

A 62-year-old male presents with an irreducible swelling and severe pain in the left groin. He had a known reducible hernia for 15 years prior to this. He had a bowel movement while in the emergency room. At surgery, a Richter’s hernia was found. Which of the following statements is TRUE?

(A) It presents lateral to the rectus sheath.

(B) It presents through the lumbar triangle.

(C) It presents through the obturator foramen.

(D) It contains a Meckel’s diverticulum.

(E) It may allow normal passage of stool.

A

(E)

In Richter’s hernia, part of the bowel wall is entrapped in the hernia sac. This results in a partial occlusion of the lumen and bowel movement remains possible, despite strangulation and gangrene of the entrapped portion.

Evaluation at surgery, of the viability of the full-bowel wall must always be carried out before returning the entrapped segment to the peritoneal cavity.

154
Q

At surgery for a right inguinal hernia, a 72-year-old man is found to have a hernia sac that is not independent of the bowel wall. The cecum forms part of the wall of the sac (Fig. 8–1). Such a hernia is properly referred to as which of the following?

(A) Incarcerated

(B) Irreducible

(C) Sliding

(D) Richter’s

(E) Interstitial

A

(C)

The term sliding refers to the peritoneum that slides along with the hernia in its passage along the cord (Fig. 8–4).

The viscus forms part of the wall of the sac. The peritoneum should not be removed from the bowel wall, because devascularization may occur.

155
Q

The following structures may be injured during surgery to repair an inguinal hernia:

(A) The ilioinguinal, genitofemoral, iliohypogastric, and lateral femoral cutaneous nerves

(B) The femoral nerve

(C) The popliteal nerve

(D) The nerve to the psoas major muscle

(E) The pudendal nerve

A

(A)

Any of these nerves may be divided, crushed, or entrapped by suture or mesh during repair of a groin hernia.

The lateral femoral cutaneous nerve is vulnerable especially during laparoscopic hernia repair.

156
Q

Which of the following structures would be encountered during repair of an inguinal hernia in a male?

(A) Spermatic cord, cremaster muscle, transversalis fascia, deep epigastric vessels, conjoined tendon

(B) Round ligament

(C) Obturator nerve

(D) Symphysis pubis

(E) Nerve to the adductor muscles of the thigh

A

(A)

All are normal and constant anatomic structures of the inguinal canal.

157
Q

In repair of a femoral hernia, the structure most vulnerable to major injury lies:

(A) Medially

(B) Laterally

(C) Anteriorly

(D) Posteriorly

(E) Superficially

A

(B)

The femoral vein lies immediately lateral to the femoral canal.

Careful attention to this structure is essential in repair of femoral herniae.

158
Q

A 28-year-old professional football player has sudden pain and swelling in the right groin when attempting to intercept a pass. He is admitted to the local emergency department. On examination, there is a tender swelling in the right groin. The scrotum and penis show no abnormality. What is the next step in management?

(A) Needle aspiration to exclude hematoma

(B) Forceful manual reduction

(C) Laparotomy within 20 minutes

(D) Preoperative preparation and exploration of the groin with hernia repair

(E) Morphine and reevaluation within 12 hours

A

(D)

Unexplained recent onset of swelling in the groin, that is not reducible, should be considered to be a strangulated inguinal or femoral hernia until proved otherwise.

Needle aspiration may cause fecal perforation and forceful manual reduction may result in the return of gangrenous bowel to the peritoneal cavity

159
Q

A 70-year-old woman presents with a tender irreducible mass immediately below and lateral to the pubic tubercle. Plain abdominal x-ray shows intestinal obstruction.

What is the likeliest diagnosis?

(A) Small-bowel carcinoma

(B) Large-bowel carcinoma

(C) Adhesions

(D) Strangulated inguinal hernia

(E) Strangulated femoral hernia

A

(E)

Strangulated femoral hernia is located below and lateral to the pubic tubercle and is more common in females.

Inguinal hernias occur in similar frequency in females, but compared to femoral hernias, they are less likely to undergo strangulation.

160
Q

Treatment with a nasogastric tube and intravenous fluids is initiated. What is the next step in treatment?

(A) Sedation to relax the patient and allow spontaneous reduction of the mass

(B) Sedation and surgery scheduled for the next elective surgical appointment

(C) Sedation and manual taxis (reduction)

(D) Emergency surgery on the left groin

(E) Emergency laparotomy for intestinal obstruction and hernia repair from the peritoneal cavity

A

(D)

This patient has a strangulated femoral hernia. Emergency surgery after appropriate resuscitation is the correct treatment.

Gangrene of strangulated bowel may be present. No attempt at manual reduction should be made, because gangrenous bowel may be returned to the peritoneal cavity.

161
Q

A 2-year-old African American boy presents with a reducible umbilical hernia, under 2-cm diameter. This is best managed by:

(A) Immediate surgery and repair with mesh

(B) Immediate surgery repair without mesh

(C) Laparoscopic repair with mesh

(D) Laparoscopic repair without mesh

(E) Periodic observation and evaluation

A

(E)

As many as 90% of umbilical herniae up to 1.5 cm in diameter will not be clinically evident by the age of 5 years.

162
Q

A 55-year-old woman, who recently had been dieting with a weight loss of 20 lb, presents with a small-bowel obstruction and pain, which radiates down the inside of her thigh to the knee. She has no past history of abdominal surgery. Which of the following is the likely diagnosis?

(A) Strangulated obturator hernia

(B) Obstructing neoplasm of the ileum

(C) Gallstone ileus

(D) Strangulated femoral hernia

(E) Fracture of the pubic bone

A

(A)

Strangulation of a bowel loop may occur at the obturator fossa, classically following weight loss, which results in loss of the fat pad that covers the area, superiorly, where the obturator membrane is deficient.

This allows entry and strangulation of a bowel loop.

Compression of the obturator nerve causes the pain down the medial side of the thigh.

CT scan with contrast may show the level of obstruction.

163
Q

A 50-year-old man presents with a complaint of a 1-cm moderately painful, tender mass situated one-third of the way between the xiphisternum and the umbilicus (Fig. 8–2). What is the most likely diagnosis?

(A) Fibrosarcoma of the abdominal wall

(B) Omphalocele

(C) Spigelian hernia

(D) Fat necrosis

(E) Epigastric hernia

A

(E)

Epigastric hernia is a defect in the linea alba between the umbilicus and the xiphisternum. It usually contains preperitoneal fat rather than omentum or bowel. It may cause pain and is commonly encountered in older patients. Sometimes it is located on either side of the midline.

Spigelian hernia occurs lateral to the linea semilunaris.

164
Q

A 70-year-old, moderately obese, male presents with a large, midline incisional hernia. One year previously, he underwent a colon resection for adenocarcinoma. Colonoscopy, metastasis workup and carcinoembryonic antigen (CEA) are normal. Which of the following statement is TRUE?

(A) Repair with mesh can be performed laparoscopically.

(B) Strangulation is uncommon because the neck is narrow.

(C) Recurrence is common, even with the use of mesh of improved quality.

(D) Surgical repair is simple to perform under local anesthesia.

(E) Patients remain very uncomfortable, even with an adequate repair.

A

(A)

Repair can be performed laparoscopically, under general anesthesia, following adequate preoperative medical preparation.

Patients are more comfortable following adequate repair.

Modern mesh is of much improved quality and recurrence has become much less common.

165
Q

Following laparoscopic preperitoneal repair of an inguinal hernia, a 50-year-old male complains of severe burning pain, which radiates down the lateral side of the ipsilateral thigh. The most likely cause is injury to which of the following:

(A) Ilioinguinal nerve

(B) Iliohypogastric nerve

(C) Genitofemoral nerve

(D) Femoral nerve

(E) Lateral femoral cutaneous nerve

A

(E)

The lateral femoral cutaneous nerve is visible in the laparoscopic approach to hernia repair. This nerve can be injured in placement of the mesh used for repair, especially if staples are used.

Great care must be taken to avoid this injury, which causes severe burning pain and paresthesia of the thigh and is very disabling.

Injury is not likely if staples are not inserted lateral to the deep inguinal ring.

166
Q

A male neonate is born with an omphalocele (shown in Fig. 8–3). This entity can be distinguished from gastroschisis, because in an omphalocele, the protrusion is:

(A) Not covered by a sac

(B) A defect in the abdominal musculature

(C) Associated with an umbilicus attached to the abdominal wall musculature

(D) Associated with partial or complete malrotation of the bowel

(E) Really contains abdominal viscera

A

(D)

In omphalocele (see Fig. 8–5), the swelling is covered by a membrane formed by the peritoneum, Wharton’s jelly, and amnion. The membrane is transparent, and underlying intestine can be seen.

The other features listed are characteristic of gastroschisis. In gastroschisis, the protrusion is not covered by a membrane and the other features listed apply.

167
Q

What is true of Spigelian hernia?

(A) It occurs exclusively in males.

(B) It involves part of the circumference of the bowel wall.

(C) It is best repaired by the classical Bassini technique of inguinal ligament repair.

(D) It occurs at the lateral edge of the linea semilunaris.

(E) It always contains the vermiform appendix.

A

(D)

Spigelian hernia occurs at the semilunar line, which extends along the lateral border of each rectus abdominis muscle.

The posterior rectus sheath is deficient at the level of the arcuate line (semicircular line) about one-third of the distance between the umbilicus and the pubic symphysis; this is the most common site for Spigelian hernia to occur through the linea semilunaris.

It occurs in both sexes.

The Bassini technique refers to inguinal hernias only.

A hernia that involves part of the bowel wall is known as a Richter’s hernia.

The appendix may or may not form part of the contents of the sac.

168
Q

A 56-year-old man is scheduled to have a left indirect hernia repaired. He is asymptomatic. Before surgical treatment, he should have which of the following?

(A) Rectal examination alone

(B) Rectal examination and sigmoidoscopy

(C) Barium enema

(D) Colonoscopy

(E) Intravenous pyelogram

A

(B) Patients who have symptoms suggestive of change in bowel habits will require a barium enema or colonoscopy.

It is important not to overlook an underlying carcinoma, which could cause the patient to strain and induce a hernia.

Carcinoma and/or polyps may be overlooked if this approach is ignored.